Difference between revisions of "1974 AHSME Problems/Problem 14"

(Created page with "==Problem== Which statement is correct? <math> \mathrm{(A)\ } \text{If } x<0, \text{then } x^2>x. \qquad \mathrm{(B) \ } \text{If } x^2>0, \text{then } x>0. </math> <math> \qqu...")
 
 
(One intermediate revision by one other user not shown)
Line 25: Line 25:
 
==See Also==
 
==See Also==
 
{{AHSME box|year=1974|num-b=13|num-a=15}}
 
{{AHSME box|year=1974|num-b=13|num-a=15}}
 +
[[Category:Introductory Algebra Problems]]
 +
{{MAA Notice}}

Latest revision as of 12:43, 5 July 2013

Problem

Which statement is correct?

$\mathrm{(A)\ } \text{If } x<0, \text{then } x^2>x. \qquad \mathrm{(B) \ } \text{If } x^2>0, \text{then } x>0.$

$\qquad \mathrm{(C) \  } \text{If } x^2>x, \text{then } x>0. \qquad \mathrm{(D) \  } \text{If } x^2>x, \text{then } x<0.$

$\qquad \mathrm{(E) \  }\text{If } x<1, \text{then } x^2<x.$

Solution

We begin by going statement by statement.

For (A), $x^2>0$ for all real $x$, and so $x^2>0>x$. Therefore, $\boxed{\text{A}}$ is true.

We can give counterexamples for all other statements:

For (B), $x=-1$

For (C), $x=-1$

For (D), $x=2$

For (E), $x=-1$

See Also

1974 AHSME (ProblemsAnswer KeyResources)
Preceded by
Problem 13
Followed by
Problem 15
1 2 3 4 5 6 7 8 9 10 11 12 13 14 15 16 17 18 19 20 21 22 23 24 25 26 27 28 29 30
All AHSME Problems and Solutions

The problems on this page are copyrighted by the Mathematical Association of America's American Mathematics Competitions. AMC logo.png